7
$\begingroup$

Let $\Omega\subset\mathbb{R}^{n}$ be an open, bounded convex domain. Denote $d_{\Omega}:\Omega\rightarrow\mathbb{R}$ the distance-to-boundary function, that is, $$ d_{\Omega}\left(x\right):=\inf\left\{ \left|x-y\right|:y\in\partial\Omega\right\} . $$ It is well-known that if $\partial\Omega$ is smooth then $d_{\Omega}$ is convex in a neighborhood of $\partial\Omega$ and $\left|\nabla d_{\Omega}\right|=1$ in this neighborhood.

I would like to reduce the smoothness assumption of the boundary. My question (suggestion) is: if $\partial\Omega$ is only Lipschitz, is it true that $d_{\Omega}$ is convex in a neighborhood of $\partial\Omega$ and $\left|\nabla d_{\Omega}\right|=1$ almost everywhere in this neighborhood?

Any reference is welcome.

$\endgroup$
1
  • $\begingroup$ I think the question means $-d_{\Omega}$, not $d_{\Omega}$. It should be well-known for the smooth case. $\endgroup$
    – Haji
    Jan 23, 2018 at 14:44

5 Answers 5

9
$\begingroup$

The proof does not require smoothness.

Assume $B(x,r_x)\subset \Omega$, $B(y,r_y)\subset \Omega$. It is sufficient to show that $$B(\tfrac{x+y}2,\tfrac{r_x+r_y}2)\subset \Omega.$$ The latter follows from convexity of $\Omega$. Indeed, the ball $B(\tfrac{x+y}2,\tfrac{r_x+r_y}2)$ is the set of midpoints between $B(x,r_x)$ and $B(y,r_y)$.

The second condition, $|\nabla \textrm{dist}_{\partial\Omega}|\stackrel{ae}{=}1$ holds for all open domains --- nothing it needed. Indeed, $\textrm{dist}_{\partial\Omega}$ is $1$-Lipschitz. Note that if $\textrm{dist}_{\partial\Omega}$ is differentiable at $x$, then $|\nabla_x \textrm{dist}_{\partial\Omega}|\le 1$. Now, by Rademacher's theorem, $\textrm{dist}_{\partial\Omega}$ is differentiable almost everywhere. Assume that $v$ be the unit vector direction in the direction of a shortest way from $x$ to the boundary. Then $(d\textrm{dist}_{\partial\Omega})(v)=-1$ therefore $|\nabla_x \textrm{dist}_{\partial\Omega}|\ge 1$.

$\endgroup$
2
  • $\begingroup$ To be specific: $|\nabla \operatorname{dist}_{\partial \Omega}(x_0)| = 1$ if the maximal ball $B(x_0, r)$ contained in $\Omega$ touches the boundary of $\Omega$ at one point only. $\endgroup$ Jan 23, 2018 at 19:12
  • $\begingroup$ @MateuszKwaśnicki I think it would help if you elaborated on why this is the case and why the set of points such that the maximal ball touches $\partial\Omega$ in a different way has measure zero. $\endgroup$
    – erz
    Jan 24, 2018 at 7:05
5
$\begingroup$

This is an extension of my comment to Anton Petrunin's answer, as requested by erz. Similar calculations are contained in most textbooks on PDEs (e.g. Gilbarg–Trudinger), and I guess one can find a detailed answer in some textbooks on convex bodies.


Claim 1: We have $|\nabla \operatorname{dist}(x_0, \partial \Omega)| = 1$ if the maximal ball $B(x_0,r)$ contained in $\Omega$ touches the boundary of $\Omega$ at one point only.

(Edited; the first version of the proof was simpler, but incomplete; the following one is much clearer if one makes a picture on the fly).

Let $x_0 \in \Omega$, let $B(x_0, r)$ be the largest ball centred at $x_0$ contained in $\Omega$, and denote by $z$ the unique common point of $\partial B(x_0,r)$ and $\partial \Omega$.

To simplify the notation, with no loss of generality suppose that $x_0$ is the origin $0$, and that $z = (0, \ldots, 0, -r)$. For $x \in \mathbb{R}^d$ we write $x = (\tilde{x}, x_n)$ with $\tilde{x} \in \mathbb{R}^{n-1}$, $x_n \in \mathbb{R}$. In particular, $z = (\tilde{0}, -r)$.

Necessarily $\Omega$ is contained in the upper half-space $\{x_n > -r\}$. Therefore, $$\operatorname{dist}(x, \partial \Omega) \leqslant x_n + r. \tag{1} $$ On the other hand, let $a = \sqrt{1 + \delta^2} > 1$ be a number close to $1$. We will later show that for $t \geqslant 0$ small enough, say $t \in [0, t_0]$, $\Omega$ contains $$B_t = B((\tilde{0}, a t), r + t) .$$ Note that these balls are tangent to the cone $$C_a = \{x \in \mathbb{R}^n : x_n > |\tilde{x}| - (a - 1) r\} .$$ Before we prove this claim, let us see how it implies Claim 1. For $x$ sufficiently close to $0$ we have $$\operatorname{dist}(x, \partial \Omega) \geqslant \operatorname{dist}(x, \partial B_t) = r + t - |x - (\tilde{0}, a t)| = r + t - \sqrt{|\tilde{x}|^2 + (x_n - a t)^2} $$ for all $t \in [0, t_0]$. If $x_n < 0$ and $|\tilde{x}| < -\delta x_n$, we choose $t = 0$ and we get $$\operatorname{dist}(x, \partial \Omega) \geqslant r - \sqrt{|\tilde{x}|^2 + x_n^2} \geqslant r - \sqrt{\delta^2 + 1} x_n = r - a |x_n| = r + a x_n. $$ Otherwise we choose $t = (a \delta)^{-1} (\delta x_n + |\tilde{x}|)$ (which is in $[0, t_0]$ if $|x|$ is small enough) and we get, after simplification, $$\operatorname{dist}(x, \partial \Omega) \geqslant r + \frac{x_n - \delta |\tilde{x}|}{a} $$ (the right-hand side is the distance to the boundary of $C_a$). Taking (1) into account, in both cases we get $$|\operatorname{dist}(x, \partial \Omega) - r - x_n| \leqslant \max\left\{a - 1, \frac{1}{a} - 1 + \frac{\delta}{a}\right\} |x| . $$ It follows that $$\limsup_{x \to 0} \frac{|\operatorname{dist}(x, \partial \Omega) - r - x_n|}{|x|} \leqslant \max\left\{a - 1, \frac{1}{a} - 1 + \frac{\delta}{a}\right\} . $$ Since $a > 1$ can be arbitrarily close to $1$, we get $$\lim_{x \to 0} \frac{|\operatorname{dist}(x, \partial \Omega) - r - x_n|}{|x|} = 0 , $$ which means that the gradient of $\operatorname{dist}(x, \partial \Omega)$ exists at $x = 0$ and it is equal to $(0, \ldots, 0, 1)$.

It remains to show that $\Omega$ contains $B_t$ for $t \geqslant 0$ sufficiently small. Note that $B_t$ is a ball tangent to the cone $C_a$, and the convex hull of $B_{t_1} \cup B_{t_2}$ contains $B_t$ for all $t \in [t_1, t_2]$. By assumption, $\Omega$ contains $B_0$, and so if $\Omega$ contains $B_{t_0}$ for some $t_0 > 0$, then it also contains $B_t$ for all $t \in [0, t_0]$. Therefore, it is sufficient to show that $\Omega$ contains $B_t$ for some $t > 0$. Suppose, contrary to this claim, that for each $t > 0$ there is a point $y_t \in B_t \setminus \Omega$. Choose any partial limit $y$ of $y_t$ as $t \to 0$. Since $y_t \notin \Omega$, we have $y \notin \Omega$. Similarly, $y_t \in \overline{B}_t$, and so $y \in \overline{B}_0$. Thus, $y \in \Omega^c \cap \overline{B}_0 = \{z\}$, that is, $y = z = (\tilde{0}, -r)$. On the other hand, $y_t \in B_t \setminus B_0$, and so the $n$-th coordinates of $y_t$ and $y$ are not less than $-a^{-1} \delta r$. But this is a contradiction, because $-r < -a^{-1} \delta r$. The proof is complete.

Claim 2: For almost every $x_0 \in \Omega$ the maximal ball $B(x_0, r)$ contained in $\Omega$ touches the boundary of $\Omega$ at a single point.

A a Lipschitz function, $f(x) = \operatorname{dist}(x, \partial \Omega)$ is differentiable almost everywhere. However, if $\partial B(x_0, r)$ has at least two points $z_1$, $z_2$ in common with $\partial \Omega$, then $f$ is not differentiable at $x_0$: by the proof of Claim 1, $\nabla f(x)$ is equal to $v_1 = (x_0 - z_1) / |x_0 - z_1|$ on $(x_0, z_1)$, and to $v_2 = (x_0 - z_2) / |x_0 - z_2|$ on $(x_0, z_2)$. Thus, if $\nabla f(x_0)$ existed, it would be equal to $v_1$ and to $v_2$ at the same time, which is impossible, because $v_1 \ne v_2$. This proves that $\nabla f(x_0)$ does not exist, and so Claim 2 follows.


By the way, is there an elementary proof of Claim 2, which does not rely on a.e. differentiability of Lipschitz functions?

$\endgroup$
4
  • $\begingroup$ I just realised that the last paragraph of the proof of Claim 1 is wrong (there may be no larger ball like that). I will try to fix it later on. $\endgroup$ Jan 24, 2018 at 12:25
  • $\begingroup$ I updated the proof of Claim 1, but I am not quite happy with the new one: it seems to be overcomplicated. Can anyone come up with a simpler solution? $\endgroup$ Jan 24, 2018 at 19:25
  • $\begingroup$ so you use convexity of the domain, right? $\endgroup$
    – erz
    Jan 28, 2018 at 21:31
  • $\begingroup$ @erz: Yes, to get (1) and to get from "all $B_t$" to "some $B_t$". I think it is in fact enough to assume that $\Omega$ satisfies the external ball condition, though. $\endgroup$ Jan 28, 2018 at 22:16
3
$\begingroup$

It is not true in general that $d_\Omega$ is convex, however smooth the boundary is. E.g., if $n\ge2$ and $\Omega$ is the unit ball centered at the origin, then $d_\Omega(x)=1-|x|$ is concave in $x\in\Omega$, and it is not convex in $x$ in any neighborhood of the boundary.

$\endgroup$
3
$\begingroup$

Here is another answer to the convexity (or more precisely concavity) part of the question, which I think is even more simple than the one Anton gave:

Let $p\in\Omega$, and $q$ be one of its closest points on $\partial\Omega$. Since $\Omega$ is convex, there exists a support hyperplane $H$ of $\Omega$ which passes through $q$. Furthermore $H$ is orthogonal to $pq$, since it supports the sphere $S_p$ centered at $p$ and passing through $q$. So $d_{\partial\Omega}(p)=d_H(p)$. Clearly $d_H(p)$ is not bigger than $d_{H'}(p)$ for any other support plane $H'$ of $\Omega$, because all these planes lie outside $S_p$. So $d_{\partial\Omega}$ is the infimum of the distance functions to support hyperplanes of $\Omega$. These functions are linear and therefore concave. So $d_{\partial\Omega}$ is concave.

$\endgroup$
1
$\begingroup$

This is not an answer to the original question, but an alternative elementary proof of Claim 1 in the answer by Mateusz Kwaśnicki. Note that it works for any open set $\Omega$.

Let $\Omega$ be an open set in $\mathbb{R}^n$. Define $d$ by $d(x)=d(x, \mathbb{R}^n\backslash\Omega)$.

Assume that $0\in \Omega$ and $z$ is the only element of $\overline{B}(0,\|z\|)\backslash\Omega$. Our goal is to show that $\nabla d (0)= -\frac{z}{\|z\|}$, i.e. $$\lim\limits_{x\to 0}\frac{d(x)-\|z\|+\left<x,\frac{z}{\|z\|}\right>}{\|x\|}=\lim\limits_{x\to 0}\left(\frac{d(x)-\|z\|}{\|x\|}+\cos\beta\right)=0,$$ where $\beta$ is the angle between $x$ and $z$.

Step 1. Let $f(x)=\|x-z\|$ and note that $d(x)\le f(x)$ and $\nabla f (0)= -\frac{z}{\|z\|}$. Hence, we only need to estimate the behaviour of $\frac{d(x)-\|z\|}{\|x\|}+\cos\beta$ from below. Also, $d(x)\ge \|z\|-\|x\|$, for every $x\in\overline{B}(0,\|z\|)$, and so $\frac{d(x)-\|z\|}{\|x\|}+\cos\beta\ge \cos\beta-1$.

Fix $\varepsilon>0$. It is clear that $\frac{d(x)-\|z\|}{\|x\|}+\cos\beta>-\varepsilon$ if $\beta<\varepsilon$. We will show that if $\beta\ge\varepsilon$ then for sufficiently small $\|x\|$ we will have $\frac{d(x)-\|z\|}{\|x\|}+\cos\beta>-2\varepsilon$.

Step 2. For every point $y$ on the sphere $\partial B(0,\|z\|)$, other than $z$ there is $\delta>0$ such that $B(y,\delta)\subset\Omega$. For $\theta\in[0,\pi]$ consider all points on this sphere whose angle with $z$ is at least $\varepsilon$. This is a compact set, and so there is $\delta>0$ such that $\delta$-thickening of this set (see the blue contour on picture 1) is contained in $\Omega$.

enter image description here

Then for every $x$ in the little blue "packman" (whose radius is $\frac{\delta}{2})$) in the centre, $d(x)$ is greater or equal to the distance from $x$ to the part of the sphere $\partial B(0,\|z\|)$, painted with the thick black. Indeed, if $w$ is a closest point in $\mathbb{R}^n\backslash\Omega$ to $x$, then the segment $[x,w]$ has to cross either the thick black part of the $\partial B(0,\|z\|)$, or the outer boundary of the picture, which should give more than $1+\frac{\delta}{2}$ (I don't know how to explain this rigorously, but it seems clear from the picture), while $\|x-z\|\le 1+\frac{\delta}{2}$.

Step 3. For any circle in $\partial B(0,\|z\|)$ and any $x$, the closest to $x$ point $w$ on the circle is the intersection of the circle with the half-space obtained from the axis of the circle and $x$ (see picture 2). Moreover if the angles between $z$ and $w$ and $z$ and $x$ are $\alpha$ and $\beta$, then $\|x-w\|^2=\|x\|^2+\|z\|^2-2\|x\|\|z\|\cos (\beta-\alpha)$, which is minimized when $\alpha$ is maximal, i.e. $\alpha=\varepsilon$. Since $d(x)\ge \|x-w\|$, using conjugates we get $$\frac{d(x)-\|z\|}{\|x\|}+\cos\beta\ge \frac{\|x\|-2\|z\|\cos (\beta-\varepsilon)}{\|z\|+\sqrt{\|x\|^2+\|z\|^2-2\|x\|\|z\|\cos (\beta-\varepsilon)}}+\cos\beta.$$ Now if $\|x\|$ is sufficiently small we have $\frac{d(x)-\|z\|}{\|x\|}+\cos\beta\ge -\cos (\beta-\theta)-\varepsilon+\cos\beta\ge -2\varepsilon.$

$\endgroup$

Your Answer

By clicking “Post Your Answer”, you agree to our terms of service and acknowledge you have read our privacy policy.

Not the answer you're looking for? Browse other questions tagged or ask your own question.